Relate between the orthocenter and h max h_\text{max}

Geometry Level 4

Let A B C ABC be an acute-angled triangle and let H H be its orthocenter. The maximum length of an altitude is 4. What is the maximum value of A H + B H + C H AH+BH+CH ?


The answer is 8.00.

This section requires Javascript.
You are seeing this because something didn't load right. We suggest you, (a) try refreshing the page, (b) enabling javascript if it is disabled on your browser and, finally, (c) loading the non-javascript version of this page . We're sorry about the hassle.

4 solutions

Calvin Lin Staff
Nov 16, 2016

Lemma: For an acute-angled triangle, H H lies within A B C ABC .

Proof: Read orthocenter .

Note: This implies that A H , B H , C H > 0 AH, BH, CH > 0 .

Lemma: Let the foot of the altitudes be D , E , F D, E, F . Then we have

A H A D + B H B E + C H C F = 2 \frac{ AH}{AD} + \frac{ BH}{BE } + \frac{ CH}{CF} = 2

Proof: Let the area of figure P Q R S PQRS be denoted by [ P Q R S ] [PQRS] .
Consider A H H D \frac{ AH}{HD} . The ratio [ A B H ] [ A B D ] = A H A D \frac{ [ABH] } { [ABD ] } = \frac{ AH } { AD} as the triangles have the same height corresponding to the same base line. Similarly, [ A C H ] [ A C D ] = A H A D \frac{ [ACH]}{ [ACD] } = \frac{ AH } { AD } . Thus, we have [ A B H C ] [ A B C ] = A H A D \frac{ [ABHC] } { [ABC ] } = \frac{ AH } { AD } .
We have similar equations for the other 2 fractions, which gives us

A H A D + B H B E + C H C F = [ A B H C ] [ A B C ] + [ B C H A ] [ A B C ] + [ C A H B ] [ A B C ] = 2 [ A B C ] [ A B C ] = 2 . \frac{ AH}{AD} + \frac{ BH}{BE } + \frac{ CH}{CF} = \frac{ [ABHC]} { [ABC] } + \frac{ [BCHA]} { [ABC] } + \frac{ [CAHB]}{[ABC]} = \frac{ 2 [ABC] } { [ABC] } = 2 . _\square

Back to the question , since A D , B E , C H 4 AD, BE, CH \leq 4 , hence we can conclude that

A H + B H + C H = A H A D × A D + B H B E × B E + C H C F × C F A H A D × 4 + B H B E × 4 + C H C F × 4 = 2 × 4 = 8 \begin{array} { l l l } AH + BH + CH & = \frac{ AH }{ AD } \times AD + \frac{ BH}{BE } \times BE + \frac{ CH}{CF} \times CF \\ & \leq \frac{ AH}{AD} \times 4 + \frac{ BH}{BE } \times 4 + \frac{ CH}{CF} \times 4 \\ & = 2 \times 4 = 8 \end{array}

Equality holds when A D = B E = C F = 4 AD = BE = CF = 4 , which occurs when we have an equialterial triangle.


As pointed out by Mark Henning, if we relax the constraints in the question to allow for right-triangles, then the above proof can be adapted in the following way.

1) Note that if A = 9 0 \angle A = 90^ \circ , then H = A H = A and so A H = 0 AH = 0 .
2) The proof of the geometry lemma still holds true. For right triangles, these fractions are either 0 or 1. However, for obtuse triangles, we have to use negative lengths to get this to work out.
3) For the inequality step, what we really want is the following:

Lemma: For a i , b i 0 a_i, b_i \geq 0 , define S = { i a i = 0 } S = \{ i | a_i = 0 \} . Then,
a i b i max i ∉ S b i × ( a i ) \sum a_i b_i \leq \max_{ i \not \in S } b_i \times ( \sum a_i )
with equality when b i b_i is a constant for i ∉ S i \not \in S .

4) Hence, in the equality case, we have to consider when the values of A H , B H , C H AH, BH, CH are 0. For non-trivial triangles, at most one of them is 0, corresponding to the right-angled triangle. In this case, equality is achieved when the other altitudes are equal, meaning we have an isosceles right triangle.

Nice, and much more geometric in flavour than my proof. Where in your proof is the fact that the maximum value does not gave to be achieved at the equilateral triangle (if we decide to let right-angled triangles be acute-angled as a borderline case)?

Mark Hennings - 4 years, 7 months ago

Log in to reply

So, the main use of acute triangles is in ensuring that 0 < A H A D < 1 0 < \frac{ AH } { AD } < 1 . Note that in the case of obtuse triangles, we would be using directed line segments, which could have negative length. For right angle triangles, the ratio is equal to 0 or 1, and we can proceed similarly. However, the inequality has to be strengthened in the following manner, to take into consideration that some of these terms are 0:

Lemma: For a i , b i 0 a_i, b_i \geq 0 , define S = { i a i = 0 } S = \{ i | a_i = 0 \} . Then, a i b i max i ∉ S b i × ( a i ) \sum a_i b_i \leq \max_{ i \not \in S } b_i \times ( \sum a_i ) with equality when b i b_i is a constant for i ∉ S i \not \in S .

Thus, in finding of the equality case, the other condition that we're missing is "If A H = 0 AH = 0 , then equality is achieved when B E = C F BE = CF ." This implies that we have an isosceles right triangle.

Calvin Lin Staff - 4 years, 7 months ago

NICE !!! I have learned a lot from these two solutions !! Thanks for posting them !

Bob Kadylo - 4 years, 7 months ago

I loved this solution

In fact, for all acute triangles,

AH + BH + CH <= 2 * h max

A Former Brilliant Member - 4 years, 5 months ago

Log in to reply

Are you sure about that?

As a gut check, consider a really obtuse angle, then the altitudes are very small but the orthocenter is very far away. This makes me think that the inequality is no longer valid. It might be valid for directed line segments, but you made no such reference to them.

Calvin Lin Staff - 4 years, 5 months ago
Mark Hennings
Nov 14, 2016

The altitude h C h_C from the vertex C C to the side A B AB has length h C = b sin A = 2 R sin A sin B h_C \,=\, b\sin A \,=\, 2R\sin A \sin B (where R R is the circumradius), with similar formulae for the other two altitudes h A , h B h_A,h_B . Thus, if we assume that the three angles A , B , C A,B,C in the acute-angled triangle are such that the angle A A is the smallest, so that 0 A B , C 1 2 π 0 \le A \le B,C \le \tfrac12\pi , then the longest altitude is h m a x = h A = 2 R sin B sin C h_\mathrm{max} \,=\, h_A \,=\, 2R\sin B \sin C . Thus we must assume that 0 A B , C 1 2 π 0 \le A \le B,C \le \tfrac12\pi , and that R sin B sin C = 2 R\sin B \sin C = 2 .

It is well known that, for an acute-angled triangle, the length A H AH is twice the distance from the circumcentre to the side B C BC , so that A H = 2 R cos A AH \,=\, 2R\cos A . Simlarly B H = 2 R cos B BH \,=\, 2R\cos B and C H = 2 R cos C CH \,=\, 2R\cos C , and hence A H + B H + C H = 2 R ( cos A + cos B + cos C ) = 4 cos A + cos B + cos C sin B sin C = 4 cos B + cos C cos ( B + C ) sin B sin C = 4 + 4 cos B + cos C cos B cos C sin B sin C = 4 + 4 ( cot B c o s e c C + c o s e c B cot C cot B cot C ) = 4 + 4 F ( B , C ) \begin{array}{rcl} AH + BH + CH & = & \displaystyle 2R(\cos A + \cos B + \cos C) \; = \; 4\frac{\cos A + \cos B + \cos C}{\sin B \sin C} \\ & = & \displaystyle 4 \frac{\cos B + \cos C - \cos(B+C)}{\sin B \sin C} \; = \; 4 + 4 \frac{\cos B + \cos C - \cos B \cos C}{\sin B \sin C} \\ & = & 4 + 4\big(\cot B \mathrm{cosec}\,C + \mathrm{cosec}\,B \cot C - \cot B \cot C\big) \; = \; 4 + 4F(B,C) \end{array} Thus we want to maximize F ( B , C ) F(B,C) over the relevant range of values of B , C B,C . Since A + B + C = π A+B+C = \pi and 0 A B , C 1 2 π 0 \le A \le B,C \le \tfrac12\pi , we deduce that π B C B , C \pi - B - C \le B,C , and hence it follows that B , C B,C must lie in the region R \mathcal{R} defined by the inequalities 2 B + C π B + 2 C π B 1 2 π C 1 2 π 2B + C \ge \pi \hspace{1cm} B + 2C \ge \pi \hspace{1cm} B \le \tfrac12\pi \hspace{1cm} C \le \tfrac12\pi Thus R \mathcal{R} is the quadrilateral region defined by the vertices ( 1 2 π , 1 4 π ) (\tfrac12\pi,\tfrac14\pi) , ( 1 2 π , 1 2 π ) (\tfrac12\pi,\tfrac12\pi) , ( 1 4 π , 1 2 π ) (\tfrac14\pi,\tfrac12\pi) and ( 1 3 π , 1 3 π ) (\tfrac13\pi,\tfrac13\pi) . Since F B = c o s e c 2 B c o s e c C [ 1 + cos B cos C cos C ] F C = c o s e c B c o s e c 2 C [ 1 + cos B cos C cos B ] \begin{array}{rcl} \displaystyle \frac{\partial F}{\partial B} & = & \displaystyle -\mathrm{cosec}^2B \mathrm{cosec}\,C\big[1 + \cos B \cos C - \cos C\big] \\ \displaystyle \frac{\partial F}{\partial C} & = & \displaystyle -\mathrm{cosec}\,B \mathrm{cosec}^2C\big[1 + \cos B \cos C - \cos B\big] \end{array} we see that F = 0 \nabla F \,=\, 0 only when 1 + cos B cos C = cos B = cos C 1 + \cos B \cos C \,=\, \cos B \,=\, \cos C . Since the quadratic equation 1 + t 2 = t 1 + t^2 = t has no real roots, we deduce that there are no points where F = 0 \nabla F = 0 . The the maximum value of F F over the region R \mathcal{R} must be achieved on the boundary of R \mathcal{R} . Since F F is a function that is symmetric in B B and C C , we only need check the behaviour on half the boundary.

  • It is clear that F ( B , 1 2 π ) = cot B F(B,\tfrac12\pi) = \cot B is a decreasing function of B B for 1 4 π B 1 2 π \tfrac14\pi \le B \le \tfrac12\pi , and so F ( B , 1 2 π ) 1 F(B,\tfrac12\pi) \le 1 for 1 4 π B 1 2 π \tfrac14\pi \le B \le \tfrac12\pi .
  • We note that F ( B , π 2 B ) = cot B c o s e c 2 B c o s e c B cot 2 B + cot B cot 2 B = 2 cos 3 B cos 2 B 2 sin 2 B cos B F(B,\pi-2B) \; = \; \cot B \mathrm{cosec}\,2B - \mathrm{cosec}\,B \cot2B + \cot B \cot 2B \; = \; \frac{2\cos^3 B - \cos 2B}{2\sin^2B \cos B} so that 1 F ( B , π 2 B ) = 2 sin 2 B cos B 2 cos 3 B + cos 2 B 2 sin 2 B cos B = cos 2 B ( 1 2 cos B ) 2 sin 2 B cos B 1 - F(B,\pi - 2B) \; =\; \frac{2\sin^2B \cos B - 2\cos^3B + \cos 2B}{2\sin^2B \cos B} \; =\; \frac{\cos2B(1 - 2\cos B)}{2\sin^2B\cos B} If 1 4 π B 1 3 π \tfrac14\pi \le B \le \tfrac13\pi , then cos 2 B 0 \cos2B \le 0 and cos B 1 2 \cos B \ge \tfrac12 , which means that F ( B , π 2 B ) 1 F(B,\pi - 2B) \,\le \, 1

We deduce that the maximum value of F ( B , C ) F(B,C) over the region R \mathcal{R} is 1 1 , and this is achieved at the points ( 1 2 π , 1 4 π ) (\tfrac12\pi,\tfrac14\pi) , ( 1 4 π , 1 2 π ) (\tfrac14\pi,\tfrac12\pi) and ( 1 3 π , 1 3 π ) (\tfrac13\pi,\tfrac13\pi) .

Thus the maximum value of A H + B H + C H AH + BH + CH , subject to the condition h m a x = 4 h_\mathrm{max} = 4 , is 8 \boxed{8} . This maximum value is achieved either by an equilateral triangle or by an isosceles right-angled triangle.

Oh wow! That's pretty intense.

Calvin Lin Staff - 4 years, 7 months ago

I gave this solution an 'Upvote' for its clarity and how much time it must have taken to 'LaTeX' it all together. Nice diagram too! You could publish Math Textbooks - or maybe you already do? I just love your work !!!!

Bob Kadylo - 4 years, 7 months ago

In Δ A B C \Delta ABC let D D and E E be the feet of altitudes on A B , B C AB , BC from vertices C , A C , A respectively and A E AE be the maximum altitude. A B , A C B C . \Rightarrow AB , AC \ge BC. Also, let B C . \angle B \ge \angle C.

Our Claim 2 A E A H + B H + C H . - 2AE \ge AH + BH + CH.

Proof 2 A E A E + C D - 2 AE \ge AE + CD . So, it suffices to prove that A F + C D A H + B H + C H H E + H D B H AF + CD \ge AH + BH + CH \Rightarrow HE + HD \ge BH

Applying Ptolemy's theorem in cyclic qua. B D H E BDHE ,

B E × H D + B D × H E = B H × D E BE \times HD + BD \times HE = BH \times DE

But C A E B H D B H B E B D \angle C \ge \angle A \Rightarrow \angle EBH \le \angle DBH \Rightarrow BE \ge BD Using in above equation:

B E × ( H E + H D ) B H × D E \Rightarrow BE \times (HE + HD) \ge BH \times DE 1 -1

But, Δ B D E Δ B C A B D E = C B B D E \Delta BDE \sim \Delta BCA \Rightarrow \angle BDE = \angle C \Rightarrow \angle B \ge \angle BDE

D E B E \Rightarrow DE \ge BE 2 - 2

Thus from 1 , 2 1 , 2 H E + H D A H . \Rightarrow HE + HD \ge AH. Thus 2 A E A H + B H + C H 2 AE \ge AH + BH + CH So, the A H + B H + C H AH + BH + CH achieves its maximum value at 8.

An another question guarateeing the uncertainty of INMO problems. :)

Rakshit Joshi
Nov 15, 2016

this solution is not correct, BUT a JEE style. since the A B C \triangle ABC is given acute and no other condition given for triangle, I considered the triangle ABC to be equilateral and then perpendiculars and centroid are in same line .

A H = 8 3 = B H = C H m a x ( A H + B H + C H ) = 8 \begin{aligned} AH= \frac{8}3 = BH = CH\\ max (AH+BH+CH) = \boxed 8 \end{aligned}

Again i have not proved triangle to be equilateral but I just let the h max to be equal and possible in the case of Equilateral triangle, Hope it helps :D . For more clarification please see @Mark hennings solution.

You have only shown that "for the equilateral triangle, the value is 8". You have not shown why this has to be the maximum value. It could be possible that with another triangle, we get a larger value.

So, even in JEE style, this isn't a solution, but just an educated guess.

Calvin Lin Staff - 4 years, 7 months ago

Log in to reply

yes sir :(, so should i remove my corrupted solution now ?

Rakshit Joshi - 4 years, 7 months ago

This problem is from INMO

Aditya Kumar - 4 years, 7 months ago

Log in to reply

oh! ok, then for that purpose you may avoid my solution :D

Rakshit Joshi - 4 years, 7 months ago

0 pending reports

×

Problem Loading...

Note Loading...

Set Loading...